1965 AHSME Problems/Problem 31

Revision as of 09:43, 19 July 2024 by Thepowerful456 (talk | contribs) (Solution)
(diff) ← Older revision | Latest revision (diff) | Newer revision → (diff)

Problem

The number of real values of $x$ satisfying the equality $(\log_ax)(\log_bx) = \log_ab$, where $a > 0, b > 0, a \neq 1, b \neq 1$, is:

$\textbf{(A)}\ 0 \qquad  \textbf{(B) }\ 1 \qquad  \textbf{(C) }\ 2 \qquad  \textbf{(D) }\ \text{a finite integer greater than 2}\qquad \textbf{(E) }\ \text{not finite}$

Solution

From the properties of logarithms, we know that $(\log_ax)(\log_xb)=\log_ab$, and $\log_xb=\frac{1}{\log_bx}$. Thus, to satisfy the given equation, we are looking for values of $x$ when $\log_bx=\frac{1}{\log_bx}$, or when $\log_bx=\pm 1$. This happens either when $x=b$ or $x=\frac{1}{b}$. Thus we have $2$ solutions, consistent with answer choice $\fbox{\textbf{(C)}}$.

See Also

1965 AHSC (ProblemsAnswer KeyResources)
Preceded by
Problem 30
Followed by
Problem 32
1 2 3 4 5 6 7 8 9 10 11 12 13 14 15 16 17 18 19 20 21 22 23 24 25 26 27 28 29 30 31 32 33 34 35 36 37 38 39 40
All AHSME Problems and Solutions

The problems on this page are copyrighted by the Mathematical Association of America's American Mathematics Competitions. AMC logo.png